User avatar
 
maryadkins
Thanks Received: 640
Atticus Finch
Atticus Finch
 
Posts: 1261
Joined: March 23rd, 2011
 
This post thanked 2 times.
 
 

Q19 - Vanwilligan: Some have argued

by maryadkins Fri Jul 15, 2011 4:14 pm

19. (D)
Question Type: Assumption


The core is:

Pros make team owners big money, so owners who are willing pay them big salaries. -->

The pros’ salaries are fair.

We are asked for the sufficient assumption"”that which, if true, would make this argument work. We need to link the key terms in our premises with the term "fair" in the conclusion, because right now, it isn’t clear why just because someone is willing to pay a certain salary, that salary is fair. (D) does this:

Pros make team owners big money, so owners who are willing pay them big salaries. -->

(Any salary that a team owner is willing to pay for the services of a professional athlete is a fair salary.) -->

The pros’ salaries are fair.

(A) tells us that the fairest economic system is one in which most goods and services are based on free market values. That doesn’t go as far as we would need"”most may not include professional athlete’s salaries. Furthermore, even in the fairest economic system, couldn’t there still be some unfair salaries? We aren’t concerned with what the fairest economic system is, but with whether these particular salaries are fair.
(B) is incorrect. If we knew that paying the pros less would mean the teams would make less, what would that tell us about how to determine whether the salaries are fair? We still don’t know what fair is based on with this answer choice.
(C) is incorrect. Again, it tells us nothing about why the salaries are fair just because people are willing (or are forced) to pay them.
(E) is incorrect and trying to fool you with reverse logic. Our critical missing piece is: willing to pay --> fair (if one is willing to pay it, it’s fair). It is not: fair-->willing to pay.

#officialexplanation
 
zana.nanic
Thanks Received: 3
Vinny Gambini
Vinny Gambini
 
Posts: 9
Joined: September 16th, 2012
 
 
 

Re: Q19 - Vanwilligan: Some have argued that

by zana.nanic Mon Oct 29, 2012 3:16 pm

In answer choice E I saw the reverse logic but the word determined confused me. Doesn't it imply causation?

Can you elaborate more on that, I always get confused by words that say things like "determined, caused etc" and cannot indentify the direction of the logic link.
User avatar
 
maryadkins
Thanks Received: 640
Atticus Finch
Atticus Finch
 
Posts: 1261
Joined: March 23rd, 2011
 
 
 

Re: Q19 - Vanwilligan: Some have argued that

by maryadkins Wed Oct 31, 2012 4:06 pm

The word "determined" does imply causation, so good catch there. But that doesn't necessarily mean that causation is the problem in the argument. Causation issues come up when you have a conclusion about how one thing is causing the other, but that's not supported by the premise. That is, you don't HAVE the causal term in the premise.

Here, the word "determined" is in the premise. We don't have a causation issue here.

The key is to look for the gap. If there is a causal claim in the conclusion that isn't supported by causation in the premise, there is a causation issue. If not, it could just be that there's one thing causing another in the argument, but that's not where the problem is.
 
hovaLSAT
Thanks Received: 0
Forum Guests
 
Posts: 12
Joined: August 22nd, 2013
 
 
 

Re: Q19 - Vanwilligan: Some have argued that

by hovaLSAT Mon Sep 02, 2013 9:03 pm

Is there any tips on how to eliminate answer choices that are the reverse of what we want? Like (E) vs. (D). in this problem.

I usually get it down to the right answer and the reverse but have a hard time eliminating the reverse.

Thanks!
 
hakopis
Thanks Received: 0
Vinny Gambini
Vinny Gambini
 
Posts: 6
Joined: June 11th, 2014
 
 
 

Re: Q19 - Vanwilligan: Some have argued that

by hakopis Wed Nov 05, 2014 3:03 pm

hovaLSAT Wrote:Is there any tips on how to eliminate answer choices that are the reverse of what we want? Like (E) vs. (D). in this problem.

I usually get it down to the right answer and the reverse but have a hard time eliminating the reverse.

Thanks!


I'm not sure whether you still care, but...

The way I ended up with (D) is like this:
We know in conditional logic that,

Logical Argument
A ---> B
Premise: A;
Conclusion: Therefore B

Now our Stimulus says:

( ) ---> ( ) missing sufficient assumption
Premise: Owner's are willing to pay big $$ (A)
Conclusion: Therefore what athletes get are fair (B)

So we need to show A --> B
In this case,
Salary Owner is willing to pay ---> Fair Salary

Any Salary Owner is willing to pay ---> Fair Salary
(A --> B)
Salary Owner is willing to pay (big$$)
(A);
Therefore it is Fair
(Therefore B)
----------------------------------------------------

If we use (E), we get:

Fair Salary --> Any Salary Owner is willing to pay
(A --> B)
Salary Owner is willing to pay (big$$)
(B)
Therefore, it is a Fair Salary
(A)

Is there a trick to figuring this out? If there is, I haven't found it.